LSAT and Law School Admissions Forum

Get expert LSAT preparation and law school admissions advice from PowerScore Test Preparation.

 Administrator
PowerScore Staff
  • PowerScore Staff
  • Posts: 8917
  • Joined: Feb 02, 2011
|
#23401
Complete Question Explanation

Parallel Flaw. The correct answer choice is (B)

The argument is that since Dr. Esposito will run in the election, and Jerome will not run against her, Dr. Esposito will be the only candidate.

The major flaw is that the stimulus ignores other possibilities. People other than Jerome might run against Dr. Esposito, so the stimulus requires the defending assumption that eliminates other potential candidates.

You can also make sure the conclusions are doubled, because the right choice must state that only one choice is available.

You should not focus too much on whether the sources are correct, because that is not as major an issue as ignoring an entire realm of possibility. Go with the more serious problem first, and save the other for choosing between Contender choices, or re-evaluating.

Answer choice (A): This argument requires the linking assumption that large stores are necessary for viable shopping districts, and so is a weak choice, because the stimulus contained no similar assumption.

Answer choice (B): This is the correct answer choice. The argumentation ignores the fact that days other than Friday and Saturday exist, so this response also requires an assumption that eliminates other options. Furthermore, this response preserves the question of truth, because we can't be certain the press release is accurate.

Answer choice (C): This argument requires the linking assumption that the interviewers will not hire someone whom they were less impressed with. However, since this choice never concludes that Marilyn must get the job, it does not make the error of ignoring other possible candidates, so this response is incorrect.

Answer choice (D): The premises in this argument actually work to limit the possibilities to Rustimann and Jones. The stimulus assumes that there are no other possibilities, but this choice states as much, so this response is not similar to the stimulus in that respect, and is incorrect.

Answer choice (E): This attractive response includes an assumption that there aren't other, better books available than the two discussed, and is similar to the stimulus in that respect. However, the argumentation in this choice also includes the linking assumption that we ought always buy what is best, and that we are beginners at programming. Since the stimulus involved no similar linking assumptions, this response is ultimately dissimilar, and incorrect.
 saranash1
  • Posts: 168
  • Joined: May 21, 2013
|
#10808
In these questions must the tense be the same in the answer choice as in the premise. Such as she will be the only candidate in the election in the premise. Must the answer choice be in future tense as well.
 Ron Gore
PowerScore Staff
  • PowerScore Staff
  • Posts: 220
  • Joined: May 15, 2013
|
#10811
Hi saranash1!

I can see you are working through these Parallel Reasoning questions, and this one is related to the one you asked about the specific homework question.

First, I assume you meant to ask whether the tense in the stimulus must be the same as in the correct answer choice, not the premise.

The bottom line for all conversation regarding Parallel Reasoning questions is that the logical structure of the argument in the correct answer must match the structure of the argument in the stimulus. Because this is a Prove family question, we can apply the Fact Test, here the Abstract Fact Test, to the answer choices. Just like a Must Be True question answer choice is incorrect if it contains some information that is supported by the stimulus and some that is not, a Parallel Reasoning question answer choice is incorrect if it only partially matches the logical structure of the argument in the stimulus, e.g., the premises match but not the conclusion.

So, for example, if in a question the abstraction of the conclusion in the stimulus is that something will not happen, a future definitive statement, then the conclusion in the correct answer choice will have to match that structure. This is what we refer to as the Double the Conclusion Test, which is part of the Parallel Reasoning Elemental Attack presented in section IX of your weekend course book.

Hope that helps.

Ron
 saranash1
  • Posts: 168
  • Joined: May 21, 2013
|
#10817
so can i conclude in number 10. on page 313 that b is correct because it says WILL be the only just as it does in the stimulus?

Whats the fact test?
 Steve Stein
PowerScore Staff
  • PowerScore Staff
  • Posts: 1153
  • Joined: Apr 11, 2011
|
#10819
Hi saranash1,

The Fact Test states that the right answer to a Must be True question can always be proven by the facts presented in the stimulus.

Similarly, in Parallel Reasoning questions, as discussed above, certain aspects of the right answer choice must parallel those presented in the stimulus--for example, the validity of an argument (or lack thereof) must parallel that of the stimulus, as well as the method of reasoning reflected.

The tense of the answer choice does not necessarily need to parallel that of the stimulus.

I hope that's helpful! Please let me know whether this is clear--thanks!

~Steve
 saranash1
  • Posts: 168
  • Joined: May 21, 2013
|
#10827
Ok. Well then what is wrong with answer choice D?
 Ron Gore
PowerScore Staff
  • PowerScore Staff
  • Posts: 220
  • Joined: May 15, 2013
|
#10851
Answer (D) does not match the reasoning in the stimulus because it has an additional premise, "these two are the only people who could have been involved." That premise reduces the universe of possible options to an either/or, in which it was only Rustimann or Jones who could have been involved. This is unlike the stimulus, in which there was no reason think the choice for candidates must be an either/or between Esposito and Krasman.

Thanks,

Ron
 saranash1
  • Posts: 168
  • Joined: May 21, 2013
|
#10911
ok that makes sense. thanks
 PositiveThinker
  • Posts: 49
  • Joined: Dec 24, 2016
|
#34875
I see how D is right but just to be clear about what the flaw is, could this be classified as a false dilemma? Thanks.
 AthenaDalton
PowerScore Staff
  • PowerScore Staff
  • Posts: 296
  • Joined: May 02, 2017
|
#34876
Hi PositiveThinker,

Yes, I this is an example of a false dilemma -- the speaker incorrectly assumes that only two candidates can run in the election, and that if Dr. Esposito runs, Jerome Krasman will not run, leaving Esposito as the only candidate in the race. Unfortunately the speaker doesn't present any information to support the assumption that the race will be limited to these two candidates, so you're right that this is a false dilemma.

Good luck,

Athena Dalton

Get the most out of your LSAT Prep Plus subscription.

Analyze and track your performance with our Testing and Analytics Package.